K
Khách

Hãy nhập câu hỏi của bạn vào đây, nếu là tài khoản VIP, bạn sẽ được ưu tiên trả lời.

28 tháng 5 2016

a) Theo định lí Pi-ta-go ta có

       AB^2+AC^2=BC^2

=> 3^2+4^2=BC^2

=> 9+16=BC^2

=> BC^2=25

=> BC=căn 25

=> BC=5

b)

Xét tam giác AMB và tam giác CMD có

AM=MC (GT)

BM=MD (GT)

Góc AMB= góc DMC (đối đỉnh)

=> tam giác AMB=tam giác CDM(cạnh-góc-cạnh)

=>góc BAM=góc MCD (=90 độ)

c)Xét tam giác vuông AMB 

Theo định lí Pi -ta-go ta có

AB^2+AM^2=BM^2

3^2+2^2=BM^2

9+4=BM^2

=>BM^2=13

=>BM=căn 13

=>2BM=2* căn 13

Mà AB+BC=3+5=8

Do 2*căn 13<8

=>2BM<8

d)chịu

28 tháng 5 2016

phần a,b,c tương đối đơn giản nên em tự chứng minh nhé

phần d : thì cũng ở mức độ khá một chút: gợi ý cho em nhé 

chứng minh: góc D = góc ABD (1)   ( vì tam giác MBA = Tam giác MDC ( c.g.c) )

xét tam giác BCD có : BC > CD ( 5cm > 3cm )=> góc D > Góc CBD hay  góc D > góc CBM (2)

Từ (1) và (2) => đpcm 

a: BC=căn 3^2+4^2=5cm

AB<AC<BC

=>góc C<góc B<góc A

b: Xét tứ giác ABCD có

M là trung điểm chung của AC và BD

=>ABCD là hình bình hành

=>AB//CD

=>CD vuông góc CA

c: CM=1/2CA=2cm

Xét ΔCBD có

CM,BN là trung tuyến

CM cắt BN tại H

=>H là trọng tâm

=>CH=2/3CM=2/3*2=4/3(cm)

d: Xét ΔDBC có

DKlà trung tuyến

H là trọng tâm

=>D,K,H thẳng hàng

a: \(BC=\sqrt{6^2+3^2}=3\sqrt{5}\left(cm\right)\)

\(BM=\sqrt{6^2+1.5^2}=\dfrac{3\sqrt{17}}{2}\left(cm\right)\)

b: Xét tứ giác ABCD có 
M là trung điểm của BD

M là trung điểm của AC

Do đó: ABCD là hình bình hành

Suy ra: AB=CD và CD//AB

hay CD\(\perp\)AC

10 tháng 5 2022

Không làm thì CÚT

8 tháng 7 2019

A B C M

CM :

a) Áp dụng định lí Pi - ta - go vào t/giác ABC vuông tại A, ta có:

BC2 = AB2 +  AC2

=> AB2 = BC2 - AC2 = 102 - 82 = 100 - 64 = 36

=> AB = 6 (cm)

b) Xét t/giác ABM và t/giác CDM

có: BM = MD (gt)

   \(\widehat{AMB}=\widehat{CMD}\) (đối đỉnh)

  AM = CM (gt)

=> t/giác ABM = t/giác CDM (c.g.c)

=> AB = CD (2 cạnh t/ứng)

=> \(\widehat{A}=\widehat{C}\) (2 góc t/ứng)

Mà \(\widehat{A}=90^0\) => \(\widehat{C}=90^0\) => AC \(\perp\)CD

c) Xét t/giác ACD

 Ta có: BC + CD > BD (bất đẳng thức t/giác)

Mà CD = AB và 2BM = BD (vì BD = BM + MD và BM = MD)

=> AB + BC > 2BM

d) Ta có: AB < BC (6 cm < 10cm)

Mà AB = CD

=> CD > BC =>  \(\widehat{MBC}< \widehat{D}\) (quan hệ giữa cạnh và góc đối diện)

Mà \(\widehat{D}=\widehat{ABM}\) (vì t/giác ABM = t/giác CDM)

=> \(\widehat{CBM}< \widehat{ABM}\)

8 tháng 3 2023

Cho tam giác ABC vuông tại A có AB<AC,đường trung tuyến AM. Trên tia đối của tia AM lấy điểm D sao cho M là trung điểm AD.

a) chứng minh tam giác MAB= tam giác MDC và DC song song với AB

b) gọi K là trung điểm AC. Chứng minh tam giác BKD cân 

c) DK cắt BC tại O. Chứng minh CO=2/3CM

d) BK cắt AD tại N. Chứng minh MK vuông góc với NO

 

a: Xét tứ giác ABCD co

M là trung điểm chung của AC và BD

=>ABCD là hình bình hành

=>AB=CD và AB//CD

=>CD vuông góc AC

b: AB+BC=AB+AD>BD=2BM

c: góc ABM=góc CDB

mà góc CDB>góc CBM

nên góc ABM>góc CBM

5:

a: ΔABC cân tại A

mà AH là trung tuyến

nên AH vuông góc BC

BH=CH=4cm

=>AH=căn 10^2-4^2=2*căn 21(cm)

b: Xét ΔIBH và ΔIAD có

góc IBH=góc IAD

IB=IA

góc BIH=góc AID

=>ΔIBH=ΔIAD

=>AD=BH=HC

 

a) Áp dụng định lí Pytago vào ΔABC vuông tại A, ta được:

\(BC^2=AB^2+AC^2\)

\(\Leftrightarrow AC^2=BC^2-AB^2=5^2-3^2=16\)

hay AC=4(cm)

Vậy: AC=4cm

a: AC=căn 5^2-3^2=4cm

b: Xét ΔMAB và ΔMCD có

MA=MC

góc AMB=góc CMD

MB=MD

=>ΔMAB=ΔMCD

=>AB=CD

c: AB+BC=CD+BC>DB=2BM(ĐPCM)